How to find tangential and normal acceleration?

Click For Summary
To find tangential and normal acceleration given the equations a_x = c*cos[d*t] and a_y = c*sin[d*t], one can utilize the provided equations for tangential and normal acceleration. The tangential acceleration can be calculated using a_t = (r*ddot{theta} + 2*dot{r}*dot{theta}) or a_t = (dv/dt)|v|, while normal acceleration is given by a_n = (ddot{r} - r*dot{theta}^2). The challenge lies in determining the appropriate values for r and theta, as they are not directly provided. To solve for the overall acceleration magnitude, use a = sqrt((a_t)^2 + (a_n)^2). Understanding that tangential acceleration is the component of acceleration in the direction of velocity is crucial for the calculations.
reslion
Messages
2
Reaction score
0

Homework Statement


The problem asks for the tangential and normal acceleration of the acceleration. We were given that:
$$a_x=c*cos[d*t]$$ and $$a_y=c*sin[d*t]$$ where c and d are constants.

Homework Equations


The book gives us
$$a_t=(r\ddot{\theta}+2\dot{r}\dot{\theta})$$, (1)
$$a_n=(\ddot{r}-r\dot{\theta}^2$$ (2)
and
$$a=\sqrt{(a_t)^2+(a_n)^2}$$ (3)
but I found online that
$$a_t=\frac{dv}{dt}|v|$$ (4).
Finally, I know that $$a=\sqrt{(a_x)^2+(a_y)^2}$$ (5).

The Attempt at a Solution


My attempt was to use equation (4) as we don't have a theta to find $$a_{tx}$$ and $$a_{ty}$$. But wouldn't that just be the given accelerations as we would integrate the acceleration just to take the derivative again?
After that I would just solve (3) for $$a_n$$ of x and y and then plug both it in the (5) to find the magnitude of both $$a_t$$ and $$a_n$$. But I'm confused on how to find the tangential with either (1) or (4)?
 
Physics news on Phys.org
I don't understand your equation 3. A correct version of that would be useful.
Remember that the tangential acceleration is, by definition, the component of the acceleration in the direction of the velocity.
 
Question: A clock's minute hand has length 4 and its hour hand has length 3. What is the distance between the tips at the moment when it is increasing most rapidly?(Putnam Exam Question) Answer: Making assumption that both the hands moves at constant angular velocities, the answer is ## \sqrt{7} .## But don't you think this assumption is somewhat doubtful and wrong?

Similar threads

  • · Replies 13 ·
Replies
13
Views
2K
  • · Replies 3 ·
Replies
3
Views
2K
Replies
6
Views
1K
Replies
5
Views
2K
Replies
4
Views
2K
  • · Replies 20 ·
Replies
20
Views
2K
  • · Replies 4 ·
Replies
4
Views
2K
Replies
8
Views
2K
  • · Replies 7 ·
Replies
7
Views
2K
  • · Replies 5 ·
Replies
5
Views
2K